Đến nội dung

Hình ảnh

$\frac{x^{3}}{3y+1}+\frac{y^{3}}{3z+1}+\frac{z^{3}}{3x+1}$


  • Please log in to reply
Chủ đề này có 4 trả lời

#1
manhhung2013

manhhung2013

    Sĩ quan

  • Thành viên
  • 306 Bài viết

x, y, z$\geqslant 0$ thỏa mãn: $x^{3}+y^{3}+z^{3}=3$. Tìm Min:

$\frac{x^{3}}{3y+1}+\frac{y^{3}}{3z+1}+\frac{z^{3}}{3x+1}$


Bài viết đã được chỉnh sửa nội dung bởi HappyLife: 24-05-2016 - 20:35

đừng nghĩ LIKE và LOVE giống nhau...
giữa LIKE và LOVE chữ cái I đã chuyển thành O,tức là Important:quan trọng đã trở thành Only:duy nhất.
chữ cái K đã chuyển thành V:Keen:say mê đã trở thành Vascurla :ăn vào mạch máu.
vì thế đừng hỏi tại sao
lim(LIKE)=LOVE nhưng lim(LOVE) =

 


#2
PlanBbyFESN

PlanBbyFESN

    Thiếu úy

  • Điều hành viên OLYMPIC
  • 637 Bài viết

x, y, z$\geqslant 0$ thỏa mãn: $x^{3}+y^{3}+z^{3}=3$. Tìm Min:

$\frac{x^{3}}{3y+1}+\frac{y^{3}}{3z+1}+\frac{z^{3}}{3x+1}$

 

Một ý tưởng AM-GM ngược dấu !


Bài viết đã được chỉnh sửa nội dung bởi PlanBbyFESN: 24-05-2016 - 20:23

:huh:


#3
Nam Duong

Nam Duong

    Trung sĩ

  • Thành viên
  • 105 Bài viết

 

AM-GM:

 

  • $\frac{x^{3}}{3y+1}=x^{3}-\frac{3x^{3}y}{3y+1}=x^{3}-\frac{3x^{3}y}{y+y+y+1}\geq x^{3}-\frac{3}{4}\frac{x^{3}y}{\sqrt[4]{y^{3}}}= x^{3}-\frac{3}{4}\sqrt[4]{x^{12}y}=x^{3}-\frac{3}{16}.4\sqrt[4]{x^{3}x^{3}x^{3}y}\geq x^{3}-\frac{3}{16}(3x^{3}+y)$

 

  • $(x^{3}+1+1)+(y^{3}+1+1)+(z^{3}+1+1)\geq 3(x+y+z)\Rightarrow x+y+z\leq 3$

 

  • $\Rightarrow \frac{x^{3}}{3y+1}+\frac{y^{3}}{3z+1}+\frac{z^{3}}{3x+1}\geq x^{3}+y^{3}+z^{3}-\frac{3}{16}(3(x^{3}+y^{3}+z^{3})+x+y+z)\geq x^{3}+y^{3}+z^{3}-\frac{3}{16}(3(x^{3}+y^{3}+z^{3})+3)=\frac{3}{4}$

 

chỗ màu đỏ xem sao  :D



#4
PlanBbyFESN

PlanBbyFESN

    Thiếu úy

  • Điều hành viên OLYMPIC
  • 637 Bài viết

Chẳng biết bài hay không mà lời giải thì khá thú vị:

 

 

x, y, z$\geqslant 0$ thỏa mãn: $x^{3}+y^{3}+z^{3}=3$. Tìm Min:

$\frac{x^{3}}{3y+1}+\frac{y^{3}}{3z+1}+\frac{z^{3}}{3x+1}$

 

$\frac{x^{3}}{3y+1}+\frac{y^{3}}{3z+1}+\frac{z^{3}}{3x+1}\geq \frac{3}{4}$

 

Cauchy-Schwarz:

 

$\frac{x^{3}}{3y+1}+\frac{y^{3}}{3z+1}+\frac{z^{3}}{3x+1}=\frac{x^{6}}{3x^{3}y+x^{3}}+\frac{y^{6}}{3y^{3}z+y^{3}}+\frac{z^{6}}{3z^{3}x+z^{3}}\geq \frac{(x^{3}+y^{3}+z^{3})^{2}}{3(x^{3}y+y^{3}z+z^{3}x)+x^{3}+y^{3}+z^{3}}=\frac{3}{1+(x^{3}y+y^{3}z+z^{3}x)}$

 

Đến đây ta cần CM: 

 

$\frac{3}{1+(x^{3}y+y^{3}z+z^{3}x)}\geq \frac{3}{4}\Leftrightarrow x^{3}y+y^{3}z+z^{3}x\leq 3$

 

Mà: $(x^{3}+x^{3}+1)+(y^{3}+y^{3}+1)+(z^{3}+z^{3}+1)\geq 3(x^{2}+y^{2}+z^{2})\Rightarrow 3\geq (x^{2}+y^{2}+z^{2})$

 

$\Rightarrow x^{3}y+y^{3}z+z^{3}x\leq 3\Leftrightarrow (x^{2}+y^{2}+z^{2})^{2}\geq 3(x^{3}y+y^{3}z+z^{3}x)$

 

Đây là 1 BĐT rất nổi tiếng và chặt của giáo sư Vasile Cirtoaje ! :)  

 

Xin trích lại 1 lời giải tự nhiên nhất:

 

 
 

 

Vì \[(a^2+b^2+c^2)^2 - 3(a^3b+b^3c+c^3a) = \frac{1}{6} \sum (a^2-2b^2+c^2+3bc-3ca)^2\] nên ta có điều phải chứng minh.

 

 

 

 

Vậy: $\frac{x^{3}}{3y+1}+\frac{y^{3}}{3z+1}+\frac{z^{3}}{3x+1}\geq \frac{3}{4}\blacksquare$


:huh:


#5
hoctrocuaHolmes

hoctrocuaHolmes

    Thượng úy

  • Thành viên
  • 1013 Bài viết

x, y, z$\geqslant 0$ thỏa mãn: $x^{3}+y^{3}+z^{3}=3$. Tìm Min:

$\frac{x^{3}}{3y+1}+\frac{y^{3}}{3z+1}+\frac{z^{3}}{3x+1}$

 

x, y, z$\geqslant 0$ thỏa mãn: $x^{3}+y^{3}+z^{3}=3$. Tìm Min:

$\frac{x^{3}}{3y+1}+\frac{y^{3}}{3z+1}+\frac{z^{3}}{3x+1}$

Áp dụng AM-GM:$x^{3}+1+1\geq 3x\Rightarrow x^{3}+3\geq 3x+1\Rightarrow \frac{x^{3}}{3x+1}\geq \frac{x^{3}}{x^{3}+3}\Rightarrow \sum \frac{x^{3}}{3y+1}\geq \sum \frac{x^{3}}{y^{3}+3}$

Đặt $x^3=a;y^3=b;z^3=c$ cho dễ nhìn :D thì ta có $a+b+c=3$

Áp dụng Cauchy-Schwarz:$\sum \frac{a^{2}}{ab+3a}\geq \frac{(a+b+c)^{2}}{\frac{(a+b+c)^{2}}{3}+3(a+b+c)}=\frac{3^{2}}{3+3.3}=\frac{3}{4}$

Dấu ''='' xảy ra khi $x=y=z=1$

 

$\frac{x^{3}}{3y+1}+\frac{y^{3}}{3z+1}+\frac{z^{3}}{3x+1}\geq \frac{3}{4}$

Cauchy-Schwarz:

$\frac{x^{3}}{3y+1}+\frac{y^{3}}{3z+1}+\frac{z^{3}}{3x+1}=\frac{x^{6}}{3x^{3}y+x^{3}}+\frac{y^{6}}{3y^{3}z+y^{3}}+\frac{z^{6}}{3z^{3}x+z^{3}}\geq \frac{(x^{3}+y^{3}+z^{3})^{2}}{3(x^{3}y+y^{3}z+z^{3}x)+x^{3}+y^{3}+z^{3}}=\frac{3}{1+(x^{3}y+y^{3}z+z^{3}x)}$

Đến đây ta cần CM: 

$\frac{3}{1+(x^{3}y+y^{3}z+z^{3}x)}\geq \frac{3}{4}\Leftrightarrow x^{3}y+y^{3}z+z^{3}x\leq 3$

Mà: $(x^{3}+x^{3}+1)+(y^{3}+y^{3}+1)+(z^{3}+z^{3}+1)\geq 3(x^{2}+y^{2}+z^{2})\Rightarrow 3\geq (x^{2}+y^{2}+z^{2})$

$\Rightarrow x^{3}y+y^{3}z+z^{3}x\leq 3\Leftrightarrow (x^{2}+y^{2}+z^{2})^{2}\geq 3(x^{3}y+y^{3}z+z^{3}x)$

Đây là 1 BĐT rất nổi tiếng và chặt của giáo sư Vasile Cirtoaje ! :)  

Phức tạp quá cậu ạ  :wacko:






1 người đang xem chủ đề

0 thành viên, 1 khách, 0 thành viên ẩn danh